Tải bản đầy đủ (.pdf) (37 trang)

Tap san Bat dang thuc

Bạn đang xem bản rút gọn của tài liệu. Xem và tải ngay bản đầy đủ của tài liệu tại đây (161.81 KB, 37 trang )

1
Tập san Bất đẳng thức
Diễn đàn Bất đẳng thức Việt Nam
VietNam Inequality Forum

Nhóm biên soạn: Ban Quản Lý VIF
• Phạm Kim Hùng [hungkhtn]
Sinh viên đại học Stanford, USA
• Nguyễn Thúc Vũ Hoàng [ZaiZai]
Học sinh chuyên Toán, niên khóa 2006-2009, THPT chuyên Lê Quý Đôn, tỉnh Quảng Trị
• Phan Phương Đức [Duca1pbc]
Học sinh chuyên Toán, niên khóa 2006-2009, THPT chuyên Phan Bội Châu, tỉnh Nghệ An
• Đinh Tuấn Đông [dtdong91]
Học sinh chuyên Toán, niên khóa 2006-2009, THPT chuyên Phan Bội Châu, tỉnh Nghệ An
• Bạch Ngọc Thành Công [10maths-tp06-09]
Học sinh chuyên Toán, niên khóa 2006-2009, THPT chuyên Trần Phú, thành phố Hải Phòng
• Lê Thúc Trình [pi3.14]
Học sinh chuyên Toán, niên khóa 2006-2009, THPT Quốc Học, thành phố Huế
• Lê Trung Kiên [kelieulinh]
Học sinh chuyên Toán, niên khóa 2004-2007, THPT chuyên Bắc Giang, tỉnh Bắc Giang
• Cùng các thành viên khác của VIF đã nhiệt tình tham gia cộng tác để hoàn thành tập san.
2
Lời giới thiệu
Diễn đàn bất đẳng thức Việt Nam đã từ lâu trở thành ngôi nhà thân thiết của rất nhiều bạn
trẻ yêu toán nói chung và bất đẳng thức nói riêng. Các bạn đã từng gặp những bất đẳng thức
với hình thức phát biểu đẹp mắt và đi cùng với những bài toán đó là những lời giải đơn giản,
trong sáng ở VIF. Các bạn muốn lưu giữ nó để làm một tài liệu tham khảo cho việc học của
mình. Các bạn muốn xem một cách có hệ thống các bài toán đã từng xuất hiện ở VIF. Hay
đơn giản là các bạn muốn có một tài liệu để học hỏi thêm kinh nghiệm. Đáp ứng nhu cầu
này chúng tôi cho ra đời tập san VIF. Đây là tập san đã được tập hợp, chọc lọc và biên tập
lại từ các bài toán đã từng xuất hiện trên VIF. Trong quá trình biên soạn không thể tránh


khỏi lỗi vì vậy mong nhận được sự bổ sung, góp ý của tất cả các bạn. Mong rằng với tài liệu
này các bạn sẽ có một tài liệu tham khảo bổ ích và tiếp tục thành công trên con đường hoa
lệ nhưng cũng không thiếu sự chông gai của: Thế giới bất đẳng thức sơ cấp.
3
Mục lục
4
Phần 1: Problems of Day
Problems of Day là các bài toán được đề xuất vào mỗi ngày của VIF. Mỗi bài toán sẽ được
đặt ra và người giải sẽ được nhận 7 hạt dẻ nếu lời giải đề xuất đúng. Đây là một box rất sôi
động của VIF. Các bài toán được chọn là các bài toán tự sáng tạo, chưa từng xuất hiện ở bất
kì nơi nào khác. Từ đây nhiều bài toán đẹp, nhiều lời giải hay đã hình thành
 Bài toán 1: [Nguyễn Thúc Vũ Hoàng]
Cho a, b, c là các số thực dương thỏa mãn abc =1. Chứng minh rằng:
1+
3

(a
3
+ 1)(b
3
+ 1)(c
3
+1)≥
1
a
+
1
b
+
1

c
Lời giải: [Phan Phương Đức]
Trong ba số a −1,b−1,c−1 luôn tồn tại 2 số cùng dấu,ta giả sử (b −1)(c −1) ≥ 0 áp dụng
bất đẳng thức Holder cho 3 số:
(a
3
+ 1)(1 + b
3
)(1 + c
3
) ≥ (a + bc)
3
Ta phải chứng minh:
a + bc +1≥ ab + bc + ca ⇔ a +1≥ ab + ca
⇔ a(1 −b)+ac(b − 1) ≥ 0
⇔ (b −1)(c −1) ≥ 0
Đẳng thức xảy ra khi và chỉ khi a = b = c =1

Lời giải 2: [Lê Thúc Trình]
Đặt a + b + c = p, ab + bc + ca = q Ta có:
(a
3
+ 1)( b
3
+ 1)( c
3
+1)=2+a
3
b
3

+ b
3
c
3
+ a
3
c
3
+ a
3
+ b
3
+ c
3
Sử dụng các kết quả sau:
a
3
+ b
3
+ c
3
= p
3
− 3pq +3
a
3
b
3
+ b
3

c
3
+ a
3
c
3
= q
3
− 3pq +3
Ta có:
(a
3
+ 1)( b
3
+ 1)( c
3
+1)=p
3
− 3pq + q
3
− 3pq +8
Bất đẳng thức cần chứng minh tương đương với:
p
3
− 3pq + q
3
− 3pq +8≥ (q − 1)
3
⇔ p
3

+3q
2
+9≥ 6pq +3q(∗)
5
Ta xét các trường hợp sau đây:
• Trường hợp 1: Nếu q ≥ p thì theo bất đẳng thức Schur ta có: p
3
+9≥ 4pq Bất đẳng thức
cần chứng minh tương đương với
3q
2
≥ 3q +2pq
Do q ≥ p nên ta phải chứng minh: q
2
≥ 3q Điều này hiển nhiên đúng.
• Trường hợp 2: Nếu p ≥ q khi đó:
(∗) ⇔ p
3
+3q
2
+9q +9≥ 6pq +12q
Theo bất đẳng thức AM-GM ta có p
3
+3q
2
+9q ≥ 9pq Như vậy ta cần chứng minh:
9pq +9≥ 6pq +12q
⇔ pq +3≥ 4q
Do p ≥ q nên pq +3≥ q
2

+3≥ 4q điều này hiển nhiên đúng vì p ≥ 3

 Bài toán 2: [Phạm Kim Hùng]
Cho a, b, c, d là các số thực không âm thỏa mãn a
2
+ b
2
+ c
2
+ d
2
=4. Chứng minh rằng:
a + b + c + d −4 ≤ 2(a −1)(b −1)(c −1)(d − 1)
Lời giải [Lê Thúc Trình]
Đặt a − 1=x, b −1=y, c − 1=z, d −1=t Điều kiện của bài toán trở thành
x
2
+ y
2
+ z
2
+ t
2
= −2(x + y + z + t)
Khi đó bất đẳng thức cần chứng minh tương đương với:
x + y + z + t ≤ 2xyzt
⇔ x
2
+ y
2

+ z
2
+ t
2
+4xyzt ≥ 0
Sử dụng bất đẳng thức AM-GM cho 4 số không âm
x
2
+ y
2
+ z
2
+ t
2
≥ 4

|xyzt|
Như vậy ta cần chứng minh

|xyzt|+ xyzt ≥ 0
Mặt khác lại có:

|xyzt| =
4

x
2
y
2
z

2
t
2
=
4

(a − 1)
2
(b − 1)
2
(c − 1)
2
(d − 1)
2
6

a
2
+ b
2
+ c
2
+4− 2(a + b + c + d)
4
=
4 − (a + b + c + d)
2
≤ 1
Như vậy ta có:
0 ≤


|xyzt|≤1
Từ đó suy ra:

|xyzt|+ xyzt ≥ 0
Dấu bằng xảy ra và chỉ khi xyzt =0hoặc xyzt = −1 Khi đó a = b = c = d =1hoặc
a = b = c =0,d=2hoặc các hoán vị

 Bài toán 3: [Phạm Kim Hùng]
Cho a, b, c, d là các số thực không âm thỏa mãn a + b + c + d =4. Chứng minh rằng:
1+ab
1+b
2
c
2
+
1+bc
1+c
2
d
2
+
1+cd
1+d
2
a
2
+
1+da
1+a

2
b
2
≥ 4
Lời giải: [Lê Thúc Trình]
Sử dụng kĩ thuật "Côsi ngược dấu". Ta có:
1+ab
1+b
2
c
2
= ab +1−
(ab +1)b
2
c
2
b
2
c
2
+1
≥ ab +1−
(ab +1)b
2
c
2
2bc
= ab +1−
(ab +1)bc
2

Tương tự với các biến còn lại, ta có:
VT ≥ 4+
(ab + bc + cd + da) − (ab
2
c + bc
2
d + cd
2
a + da
2
b
2
Như vậy ta cần chứng minh:
(ab + bc + cd + da) − (ab
2
c + bc
2
d + cd
2
a + da
2
b
2
≥ 0
⇔ ab + bc + cd + da ≥ (ab + cd)(ac + bd)
Sử dụng bất đẳng thức AM-GM ta có
ab + bc + cd + da ≥ 2

(ab + cd)( ac + bd)
Mặt khác do

ab + bc + cd + da ≤ 4
Nên ta suy ra
0 ≤

(ab + cd)(ac + bd) ≤ 2
7
Từ đó
2

(ab + cd)( ac + bd) ≥ (ab + cd)(ac + bd)
Bài toán đã đượ c chứng minh. Đẳng thức xảy ra khi và chỉ khi a = b = c = d =1hoặc
a = b = c =0,d=4hoặc các hoán vị

 Bài toán 4: [Phạm Kim Hùng]
Cho a, b, c là các số thực không âm. Chứng minh rằng:
a
1+ab
+
b
1+bc
+
c
1+ca
≤ a + b + c −
3abc
1+
3

a
2

b
2
c
2
• Lời giải 1: [Bạch Ngọc Thành Công]
Bất đẳng thức cần chứng minh tương đương với:


cyc
a
2
b
1+ab

3abc
1+
3

a
2
b
2
c
2
Ta đặt
a =
kx
y
,b=
ky

z
,c =
kz
x
Khi đó bất đẳng thức tương đương với:

cyc
k
3
x
2
y(k
2
x + z)

3k
3
1+k
2
Sử dụng bất đẳng thức Cauchy-Schwars, ta có:
VT ≥
k
3
(x + y + z)
2
(k
2
+ 1)( xy + yz + zx)

3k

3
k
2
+1
• Lời giải 2: [Phan Phương Đức]
Bất đẳng thức cần chứng minh tương đương với:

a
2
b
1+ab

3abc
1+
3

a
2
b
2
c
2


a
c + abc

3
1+
3


a
2
b
2
c
2
Sử dụng bất đẳng thức Cauchy-Schwars, ta có:
VT ≥
(a + b + c)
2
ab + bc + ca + abc(a + b + c)

3
1+
3

a
2
b
2
c
2
8
Điều này hiển nhiên đúng khi ta qui đồng mẫu số và biến đổi.

 Bài toán 5: [Phạm Kim Hùng]
Cho a
1
,a

2
, , a
n
là các số thực không âm có tổng bằng 1.Tìm giá trị lớn nhất của:
P = a
4
1
(1 − a
1
)+a
4
2
(1 −a
2
)+ + a
4
n
(1 − a
n
)
Lời giải: [Phạm Sinh Tân]
Không mất tính tổng quát giả sử
a
1
≤ a
2
≤ a
3
≤ ≤ a
n

Ta đặt
f(a
1
,a
2
,a
3
, a
n
)=
1
12
(

a
i
)
5


a
4
1
(a
2
+ a
3
+ + a
n
)

Như vậy ta phải chứng minh:
f(a
1
,a
2
, a
n
) ≥ f(0,a
1
+ a
2
,a
3
, a
n
)
⇔ a
4
1
(a
1
+ a
2
)+a
4
2
a
1
≤ 2a
1

a
2
(2a
2
1
+3a
1
a
2
+2a
2
2
)(

a
3
)
Điều này hiển nhiên đúng vì:
a
1
≤ a
2
≤ a
3
≤ ≤ a
n
Tiếp tục quá trình như trên ta chứng minh được:
f(a
1
,a

2
, a
n
) ≥ f(0,a
1
+ a
2
,a
3
, a
n
) ≥ f(0, 0, 0, , a
1
+ a
2
+ a
n−2
,a
n−1
,a
n
)
Ta đặt:
a
1
+ a
2
+ a
n−2
= x, a

n−1
= y,a
n
= z
Ta sẽ phải chứng minh:
1
12
(x + y + z)
5
≥ x
4
(y + z)+y
4
(x + z)+z
4
(x + y)
Bất đẳng thức này khá quen thuộc, có thể sử dụng phương pháp dồn biến để chứng minh
như sau, đặt:
f(x, y, z)=x
4
(y + z)+y
4
(x + z)+z
4
(x + y) −
1
12
(x + y + z)
5
Giả sử x ≥ y ≥ z Ta cần chứng minh:

f(x, y, z) −f(x, y + z, 0) = (2x(2y
2
+3yz +2z
2
) −y
3
−z
3
)yz ≥ 0
9
Không mất tính tổng quát, ta chuẩn hóa a + b + c =1, khi đó:
(x, 1 −x, 0)=3x
4
− 6x
3
+4x
2
− x +
1
12
f

(x, 1 −x, 0) =
(2x − 1)(6x +

3 −3)(6x −

3 −3)
6
≥ 0

Đẳng thức xảy ra khi và chỉ khi: (x, y, z) = ((2 −

3)a, a, 0) hoặc (x, y, z) = ((2 +

3)a, a, 0)

 Bài toán 6: [Phạm Kim Hùng]
Cho a, b, c, k là các số thực không âm. Chứng minh rằng:
3
k
2
+ k +1

a
ka + k
2
b + c
+
b
kb + k
2
c + a
+
c
kc + k
2
a + b

1
k

Lời giải: [Phan Phương Đức]
Sử dụng bất đẳng thức Cauchy-Schawrz ta có:

a
ka + k
2
b + c

(a + b + c)
2
k(a
2
+ b
2
+ c
2
)+k
2
(ab + bc + ca)+ab + bc + ca

3
k
2
+ k +1
Bất đẳng thức cuối tương đương với:
(k − 1)
2
((a − b)
2
+(b −c)

2
+(c − a)
2
) ≥ 0
Điều này hiển nhiên đúng. Lại có:

(
a
ka + k
2
b + c

a
(a + b + c)k
) ≤ 0


(k − 1)(ac −kab)
ka + k
2
b + c
≤ 0
⇔ (k −1)(
(ac − kab)
ka + k
2
b + c
+
(ba − kbc)
kb + k

2
c + a
+
(cb − kca)
kc + k
2
a + b
) ≤ 0
Ta sẽ chia bài toán ra 2 trường hợp:
• Trường hợp 1: k ≥ 1

(ac − kab)
ka + k
2
b + c
+
(ba − kbc)
kb + k
2
c + a
+
(cb − kca)
kc + k
2
a + b


ac − ab
a + b + c
=0

Khi đó bất đẳng thức đúng.
• Trường hợp 2: k ≤ 1 ta chỉ cần xét ngược lại. Bài toán được chứng minh hoàn toàn.

10
 Bài toán 7: [Nguyễn Thúc Vũ Hoàng]
Cho a, b, c, d là các số thực không âm thỏa mãn a
2
+ b
2
+ c
2
+ d
2
=4. Chứng minh rằng:
1
5 − ab
+
1
5 −bc
+
1
5 −cd
+
1
5 − da
≤ 1
Lời giải: [Bạch Ngọc Thành Công]
Giả sử: a ≤ c, b ≤ d. Ta đặt: ab = x, bc = y, cd = z, da = t. Bất đẳng thức cần chứng minh
tương đương với:
1

5 −x
+
1
5 −y
+
1
5 −z
+
1
5 −t
≤ 1

(1 − x)(1 + x)
(5 − x)(1 + x)
+
(1 − y)(1 + y)
(5 − y)(1 + y)
+
(1 − z)(1 + z)
(5 − z)(1 + z)
+
(1 − t)(1 + t)
(5 − t)(1 + t)
≥ 0
Ta xét 2 trường hợp:
• Trường hợp 1: x ≤ y ≤ t ≤ z Ta dễ dàng chứng minh được:
(1 − x)(1 + x) ≥ (1 −y)(1 + y) ≥ (1 − t)(1 + t) ≥ (1 − z)(1 + z)
(5 − x)(1 + x) ≤ (5 −y)(1 + y) ≤ (5 − t)(1 + t) ≤ (5 − z)(1 + z)
Sử dụng bất đẳng thức Chebyshev ta có:
VT ≥ [(1 −x)(1 + x)+(1−y)(1 + y)+(1− t)(1 + t)+(1−z)(1 + z)]

×

1
(5 − x)(1 + x)
+
1
(5 − y)(1 + y)
+
1
(5 − t)(1 + t)
+
1
(5 −z)(1 + z)

• Trường hợp 2: x ≤ t ≤ y ≤ z Ta cũng dễ dàng chứng minh được:
(1 − x)(1 + x) ≥ (1 −t)(1 + t) ≥ (1 − y)(1 + y) ≥ (1 − z)(1 + z)
(5 − x)(1 + x) ≤ (5 −t)(1 + t) ≤ (5 − y)(1 + y) ≤ (5 − z)(1 + z)
Sử dụng bất đẳng thức Chebyshev ta có:
VT ≥ [(1 −x)(1 + x)+(1−t)(1 + t)+(1− y)(1 + y)+(1−z)(1 + z)]
×

1
(5 − x)(1 + x)
+
1
(5 − t)(1 + t)
+
1
(5 − y)(1 + y)
+

1
(5 −z)(1 + z)

Cuối cùng ta chỉ cần chứng minh:
[(1 − x)(1 + x)+(1−t)(1 + t)+(1− y)(1 + y)+(1−z)(1 + z)] ≥ 0
⇔ 4 ≥ (a
2
+ c
2
)(b
2
+ d
2
)
11
Điều này hiển nhiên đúng theo bất đẳng thức AM-GM.
Lời giải 2 [Nguyễn Thúc Vũ Hoàng]
Bất đẳng thức được chứng minh bằng cách cộng các bất đẳng thức phụ sau đây:
1
5 − ab

(ab)
2
+7
32
,
1
5 −bc

(bc)

2
+7
32
,
1
5 −cd

(cd)
2
+7
32
,
1
5 − ab

(da)
2
+7
32
Chú ý rằng:
1
5 − ab

(ab)
2
+7
32
⇔ 0 ≤
(3 −ab)(ab −1)
2

32(5 −ab)
Điều này hiển nhiên đúng vì: 2ab ≤ a
2
+ b
2
≤ a
2
+ b
2
+ c
2
+ d
2
=4⇒ ab ≤ 2 Tương tự với
các biến còn lại. Như vậy ta cần chứng minh:
a
2
b
2
+ b
2
c
2
+ c
2
d
2
+ d
2
a

2
≤ 4
⇔ (a
2
+ c
2
)(b
2
+ d
2
) ≤
(a
2
+ b
2
+ c
2
+ d
2
)
2
4
=4
Bài toán được chứng minh.

 Bài toán 8: [Bạch Ngọc Thành Công]
Cho a, b, c là 3 cạnh của 1 tam giác . Chứng minh rằng:
b
a
+

c
b
+
a
c
+3≥ 2

a + b
a + c
+
b + c
b + a
+
c + a
c + b

Lời giải: [Lê Thúc Trình]

12
Phần 2: Liberal Match
 Bài toán 8: [Lê Trung Kiên]
Cho a,b,c là các số thực không âm. Chứng minh rằng::

a +2b
a +2c
+

b +2c
b +2a
+


c +2a
c +2b
≥ 3
Lời giải 1: [Lê Trung Kiên]
Ta đặt:
x =

a +2b
a +2c
,y =

b +2c
b +2a
,z =

c +2a
c +2b
Từ đó ta có:
1
x
2
+1
+
1
y
2
+1
+
1

z
2
+1
=
3
2
Dùng phản chứng ta sẽ qui bài toán trên về chứng minh bài toán sau: Cho x, y, z là các số
thực không âm thỏa mãn x + y + z =3. Chứng minh rằng::
f(x, y, z)=
1
x
2
+1
+
1
y
2
+1
+
1
z
2
+1

3
2
Bài toán này khá đơn giản và sau đây xin giới thiệu 2 cách như sau:
• [Lê Trung Kiên] Không mất tính tổng quát giả sử
c = max (a, b, c) → y + z ≥ 1+
1


3
,x ≤ 1
Dễ dàng chứng minh:
f(x, y, z) ≥ f(x,
y + z
2
,
y + z
2
)=
1
x
2
+1
+
8
(3 − x)
2
+4

3
2
Vậy bài toán được chứng minh. Đẳng thức xảy ra khi và chỉ khi a = b = c

• [Nguyễn Thúc Vũ Hoàng] Ta sử dụng bất đẳng thức phụ sau:
1
x
2
+1


2 − x
2

x(x −1)
2
2(x
2
+1)
≥ 0
Tương tự ta cũng có các bất đẳng thức phụ sau:
1
y
2
+1

2 −y
2
,
1
z
2
+1

2 −z
2
13
Cộng các bất đẳng thức trên lại ta có điều phải chứng minh.

Lời giải 2: [Phan Phương Đức] Không mất tính tổng quát, chuẩn hóa a + b + c =1. Ta đặt:

a +2b = x, b +2c = y,c +2a = z → x + y + z =3
Khi đó bất đẳng thức cần chứng minh tương đương với:

x
2 − x
+

y
2 − y
+

z
2 −z
≥ 3
Sử dụng bất đẳng thức Holder ta có:
N
2
× A ≥ (x + y + z)
3
=27
Trong đó:
N =

x
2 −x
+

y
2 −y
+


z
2 − z
A = x
2
(2 −x)+y
2
(2 −y)+z
2
(2 − z)
Từ đó ta cần chứng minh:
3+x
3
+ y
3
+ z
3
≥ 2(x
2
+ y
2
+ z
2
)
⇔ (x + y + z)
3
+9(x
3
+ y
3

+ z
3
) ≥ 6(x + y + z)(x
2
+ y
2
+ z
2
)
⇔ 4(x
3
+ y
3
+ z
3
)+6xyz ≥ 3xy(x + y)+3yz(y + z)+3zx(z + x)
Điều này hiển nhiên đúng theo bất đẳng thức Schur và AM-GM

 Bài toán : [Phạm Kim Hùng] Cho a,b,c,d là các số thực dương thỏa mãn a+b+c+d=4.
Chứng minh rằng:
1
6 − ab
+
1
6 −bc
+
1
6 −cd
+
1

6 − da
≤ 1
Lời giải [Lê Thúc Trình]
Đưa bất đẳng thức trên về dạng:
12 −(ab + cd)
36 −6(ab + cd)+abcd
+
12 −(bc + ad)
36 −6(bc + ad)+abcd
≤ 1
14
Mặt khác ta lại có:
12 −(ab + cd)
36 −6(ab + cd)+abcd
+
12 −(bc + ad)
36 −6(bc + ad)+abcd

12 −x
36 −6x
+
12 −y
36 −6y
Trong đó x = ab + cd, y = bc + ad. Như vậy ta cần chứng minh:
12 −x
36 −6x
+
12 −y
36 −6y
≤ 1


12 −x
6 − x
+
12 −y
6 − y
≤ 6
⇔ 144 − 18(x + y)+2xy ≤ 216 −36(x + y)+6xy
⇔ 72 − 18(x + y)+4xy ≥ 0
Điều này hiển nhiên đúng vì x + y ≤ 4 do a + b + c + d =4. Đẳng thức xảy ra khi và chỉ khi
a = b =0,c= d =2hoặc các hoán vị

 Bài toán : [Phạm Kim Hùng]
Cho a,b,c là các số thực dương thỏa mãn a
2
+ b
2
+ c
2
=3. Chứng minh rằng:

2

a + b
−1

2

b + c
− 1


2

c + a
− 1




2 −1

3
Lời giải: [Lê Trung Kiên] Ta đặt
f(a, b, c)=

2

a + b
− 1

2

b + c
− 1

2

c + a
− 1


Ta sẽ chứng minh
f(a, b, c) ≥ f (a, t, t),t=

b
2
+ c
2
2
Trước hết dễ thấy
2

b + c
− 1 ≥

2
t
− 1
Do đó ta cần phải chứng minh:
(
2

a + b
− 1)(
2

c + a
− 1) ≥ (
2

a + t

−1)
2

(2 −

a + c)(2 −

a + b
(2 −

a + t)
2


(a + c)(a + b)
a + t
15

2(2t − b − c) −(a + t −

(a + b)(a + c))
(2 −

a + t)
2
+
(a + t −

(a + b)(a + c)
(a + t)(a + t +


(a + c)(a + b))
Dễ thấy
a + t −

(a + b)(a + c) ≥ 0
Ta phải chứng minh:
2(2t −b −c) −(a + t −

(a + b)(a + c) ≥ 0
⇔ (b −c)
2

2
2t + b + c

2t +2a + b + c
2(2t + b + c)(a + t +

(a + b)(a + c))

≥ 0
⇔ 2(a + t ) −b − c +

(a + b)(a + c) ≥ 0
Điều này hiển nhiên đúng. Vậy ta kết luận:
f(a, b, c) ≥ f(a, t, t) ⇒ f (a, b, c) ≥ f(1, 1, 1) = (

2 − 1)
3


 Bài toán : [Phạm Kim Hùng] Cho a,b,c,d là các số thực dương thỏa mãn a +b + c + d =2.
Chứng minh rằng:
(a
2
+ b
2
+ c
2
)(b
2
+ c
2
+ d
2
)(c
2
+ d
2
+ a
2
)(d
2
+ a
2
+ b
2
) ≤ 4
Lời giải: Ta đặt
N(a, b, c, d)=(a

2
+ b
2
+ c
2
)(a
2
+ b
2
+ d
2
)(a
2
+ c
2
+ d
2
)(b
2
+ c
2
+ d
2
)
Sử dụng bất đẳng thức quen thuộc sau:
A
2
+ B
2
≤ (A + B)

2
Từ đó ta có:
(a
2
+ c
2
+ d
2
)(b
2
+ c
2
+ d
2
) ≤ [a
2
+(c + d)
2
][b
2
+(c + d)
2
]
Lại có
(a
2
+ b
2
+ c
2

)(a
2
+ b
2
+ d
2
) ≤ (a
2
+ b
2
)[a
2
+ b
2
+(c + d)
2
]
Từ đó suy ra:
N(a, b, c, d) ≤ N(a, b, c + d, 0)
Tương tự ta chứng minh được:
N(a, b, c + d) ≤ N(a, b + c + d, 0)
16
Bài toán qui về tìm cực trị của biểu thức sau
f(x)=
256x
2
(x
2
+1)
2

(x +1)
8
≤ max[f(1),f(0)] = 4
Đẳng thức xảy ra khi và chỉ khi a = b =1,c = d =0

 Bài toán : [Phạm Kim Hùng]
Cho a,b,c là các số thực âm. Chứng minh rằng:
b + c

a
2
+ bc
+
c + a

b
2
+ ca
+
a + b

c
2
+ ab
≥ 4
Lời giải: [Nguyễn Dũng] Sử dụng bất đẳng thức Holder ta có:
A
2
× B ≥ 8(a + b + c)
3

Trong đó:
A =
b + c

a
2
+ bc
+
c + a

b
2
+ ca
+
a + b

c
2
+ ab
B =(b + c)(a
2
+ bc)+(c + a)(b
2
+ ca)+(a + b)(c
2
+ ab)
Ta qui bài toán về chứng minh:
(a + b + c)
3
≥ 2[(b + c)(a

2
+ bc)+(c + a)(b
2
+ ca)+(a + b)(c
2
+ ab)]
Bất đẳng thức trên tương đương với:
a
3
+ b
3
+ c
3
+6abc ≥ (a
2
b + b
2
c + c
2
a)+(ab
2
+ bc
2
+ ca
2
)
Điều này hiển nhiên đúng theo bất đẳng thức Schur và vì 3abc ≥ 0 Đẳng thức xảy ra khi và
chỉ khi a=b,c=0 hoặc các hoán vị

 Bài toán : [Bạch Ngọc Thành Công]

Cho a,b,c là các số thực dương. Chứng minh rằng:
a
2
b
+
b
2
c
+
c
2
a
≥ 3
5

a
5
+ b
5
+ c
5
3
Lời giải: [Bạch Ngọc Thành Công] Ta có một bổ đề sau:

a
2
b
+
b
2

c
+
c
2
a

2
≥ 3

a
3
b
+
b
3
c
+
c
3
a

17
Lại có theo bất đẳng thức Vasile Cirtoaje

a
3
b
+
b
3

c
+
c
3
a

2
≥ 3


a
9
c
+

b
9
a
+

c
9
a

Sử dụng bất đẳng thức Holder ta có:


a
9
c

+

b
9
a
+

c
9
a

5
(

a
15
c
5
+

b
15
a
5
+

c
15
a
5

) ≥ (a
5
+ b
5
+ c
5
)
6
Sử dụng bất đẳng thức Vasile Cirtoaje một lần nữa thì ta có:
(a
5
+ b
5
+ c
5
)
2
≥ 3(

a
15
c
5
+

b
15
a
5
+


c
15
a
5
)
Bài toán được chứng minh

18
Death match
 Bài toán : [Bạch Ngọc Thành Công]
Cho a,b,c là các số thực dương. Chứng minh rằng:

a
3
b
2
+
b
3
c
2
+
c
3
a
2

2
+


2

13
4
− 1

(a+b+c)
2

13
4

a
4
b
2
+
b
4
c
2
+
c
4
a
2

+


6

13
4

13
4

(a
2
+b
2
+c
2
)
Lời giải: [Bạch Ngọc Thành Công] Giả sử c=max(a,b,c). Đưa bất đẳng thức cần chứng minh
về dạng sau:
f(a, b, c)=M(a −b)
2
+ N (a − c)(b −c) ≥ 0
Trong đó ta có:
M =(

cyc
a
3
b
2
+


a)
(a + b)(a
2
+ ab + b
2
)
a
2
b
2

13(a + b)
2
(a
2
+ b
2
)
4a
2
b
2
− 2

13
N =(

cyc
a
3

b
2
+

a)
(a + c)(b
2
+ bc + c
2
)
a
2
c
2

13(a + c)(b + c)(b
2
+ c
2
)
4a
2
c
2
−2

13
Công việc còn lại của chúng ta là chứng minh M,N ≥ 0. Điều này được chứng minh như sau:
• M ≥ 0 Ta chứng minh được 2 bổ đề sau:


cyc
a
2
b
≥ 3
3

a
3
+ b
3
+ c
3
3
≥ 3
3

a
3
+ b
3
2
a
2
+ ab + b
2
≥ 3
3

(a + b)

4
(a
2
+ b
2
)
32
Vậy ta có:


cyc
a
3
b
2
+

a

(a + b)(a
2
+ ab + b
2
)


2

cyc
a

2
b

(a + b) ×3
3

(a + b)
4
(a
2
+ b
2
)
32
≥ 18(a + b)
2
×
3

(a
2
+ b
2
)(a
3
+ b
3
)(a + b)
64
≥ 18(a + b)

2
×
3

(a
2
+ b
2
)
3
4
3
=
18
4
(a
2
+ b
2
)(a + b)
2
Vậy ta chứng minh được M ≥ 0
• N ≥ 0 Sử dụng bất đẳng thức AM-GM ta có:
a
3
3b
2
+
b
3

+
b
3
≥ a
19
c
3
a
2
+ a + a ≥ 3c


cyc
a
3
b
2
+

a ≥
2a
3
3b
2
+
b
3
c
2
+

b
3
+4c
Lại có:
2a
2
c
2
≤ (a + c)

a
3
c
3
Vậy:
N ×
a
2
c
2
a + c
≥ (
2a
3
3b
2
+
b
3
c

2
+
b
3
+4c)(b
2
+ bc + c
2
) −
13
4
(b + c)(b
2
+ c
2
) −

13

a
3
c
3
=
2a
3
3
+
2a
3

c
3b
+
2a
3
c
2
3b
2
+
b
4
c
+
b
5
c
2
+
13
12
b
2
c +
13
12
bc
2
+
3

4
c
3

23
12
b
3


13

a
3
c
3
Theo bất đẳng thức AM-GM:
b
4
c
+(
23
48
)
2
b
2
c ≥
23
24

b
3
b
5
c
2
+(
23
48
)
2
bc
2

23
24
b
3
Lại có:
2a
3
c
3b
+(
13
12
− (
23
48
)

2
)bc
2
≥ A ×

a
3
c
3
2a
3
c
2
3b
2
+(
13
12
− (
23
48
)
2
)b
2
c ≥ B ×

a
3
c

3
2a
3
3
+
3c
3
4
≥ C ×

a
3
c
3
Bằng tính toán và cộng các bất đẳng thức trên lại, ta có N ≥ 0. Bài toán được chứng minh.
• Ta có bài toán mạnh hơn bài toán trên như sau:
Cho a,b,c là các số thực dương. Chứng minh rằng:

a
3
b
2
+
b
3
c
2
+
c
3

a
2

2
+

2

19
4
− 1

(a+b+c)
2

13
4

a
4
b
2
+
b
4
c
2
+
c
4

a
2

+

6

19
4

13
4

(a
2
+b
2
+c
2
)

 Bài toán : [Phạm Kim Hùng]
Cho a,b,c là các số thực dương. Chứng minh rằng:
(a
4
+ b
4
+ c
4
)(ab + bc + ca) ≥ (a

2
+ b
2
+ c
2
)(a
2
b
2
+ b
2
c
2
+ c
2
a
2
)
20
Lời giải 1: [Bạch Ngọc Thành Công] Giả sử c=min(a,b,c). Đưa bất đẳng thức về dạng:
M(a −b)
2
+ N (a − c)(b −c) ≥ 0
Trong đó:
M =
(a + b)
2
a
2
b

2
+ b
2
c
2
+ c
2
a
2

1
ab + bc + ca
N =
(a + c)(b + c)
a
2
b
2
+ b
2
c
2
+ c
2
a
2

1
ab + bc + ca
Ta có:

(a + b)
2
(ab + bc + ca) ≥ (ab + bc + ca)
2
≥ a
2
b
2
+ b
2
c
2
+ c
2
a
2
⇒ M ≥ 0
(a + c)(b + c)(ab + bc + ca) ≥ (ab + bc + ca)
2
≥ a
2
b
2
+ b
2
c
2
+ c
2
a

2
⇒ N ≥ 0
Lời giải 2: [Nguyễn Thúc Vũ Hoàng] Đặt p = a + b + c, q = ab + bc + ca, r = abc. Khi đó bất
đẳng thức cần chứng minh tương đương với:
(p
4
−4p
2
q +2q
2
+4pr)q ≥ (p
2
− 2q)(q
2
− 2pr)
Không mất tính tổng quát chuấn hóa q = ab + bc + ca =1. Khi đó ta cần chứng minh:
p
4
− 4p
2
+2+4pr ≥ p
2
−2p
3
r −2+4pr
Điều này tương đương với:
(p
4
− 5p
2

+4+6pr)+(2p
3
r −6pr ) ≥ 0
Theo bất đẳng thức Schur bậc 4 thì:
(p
4
− 5p
2
+4+6pr) ≥ 0
Lại có:
2p
3
r − 6pr =2pr (p
2
− 3) ≥ 0
Điều này hiển nhiên đúng do:
p
2
− 3=(a + b + c)
2
− 3(ab + bc + ca)=(a −b)
2
+(b −c)
2
+(c − a)
2
≥ 0
Lời giải 3: [Lê Trung Kiên] Bất đẳng thức cần chứng minh:
a
4

+ b
4
+ c
4
a
2
b
2
+ b
2
c
2
+ c
2
a
2

a
2
+ b
2
+ c
2
ab + bc + ca

(a
2
− b
2
)

2
+(c
2
− a
2
)
2
+(b
2
− c
2
)
2
(a − b)
2
+(b − c)
2
+(c − a)
2

a
2
b
2
+ b
2
c
2
+ c
2

a
2
ab + bc + ca
≥ 0
21
Đặt VT = f(a, b, c). Sử dụng phương pháp dồn biến toàn miền chứng minh được
f(a, b, c) ≥ f (a −x, b −x, c −x)[0 ≤ x ≤ min(a, b, c)]
Từ đó có thể giả sử có một biến bằng 0, bất đẳng thức đưa về hiển nhiên:
ab(a
4
+ b
4
) ≥ a
2
b
2
(a
2
+ b
2
)
Có thể hiểu đơn giản hơn mà không cần sử dụng đến dồn biến toàn miền bằng cách chứng
minh:
(a
2
− b
2
)
2
+(c

2
− a
2
)
2
+(b
2
− c
2
)
2
(a −b)
2
+(b − c)
2
+(c − a)
2

(a
2
− b
2
)
2
+ a
4
+ b
4
(a −b)
2

+ b
2
+ a
2
a
2
b
2
+ b
2
c
2
+ c
2
a
2
ab + bc + ca
≤ ab
Hai điều này được chứng minh khá đơn giản.

 Bài toán : [Phạm Kim Hùng]
Cho a,b,c,d là các số không âm thỏa mãn a+b+c+d=3. Chứng minh rằng:
ab(a +2b +3c)+bc(b +2c +3d)+cd(c +2d +3a)+da(d +2a +3b) ≤ 6

3
Lời giải: [Lê Trung Kiên]
Đặt VT = f(a, b, c, d) Đầu tiên ta chứng minh
f(a, b, c, d) ≤ f(a, b, c + d, 0)
Điều này khá đơn giản. Tiếp theo ta phải chứng minh bài toán sau:
Cho a, b, c ≥ 0 thỏa mãn a + b + c =3. Chứng minh :

f(a, b, c)=ab(a +2b +3c)+bc(b +2c)=b
2
(2a + c)+b(a +2c)(a + c) ≤ 6

3
Ta sẽ chứng minh như sau. Xét hai trường hợp sau:
• Trường hợp 1: b ≥ c + a. Khi đó ta có:
f(a, b, c) − f(a + c, b, 0) = bc(c + a −b) ≤ 0 ⇒ f(a, b, c) ≤ 2b
2
(c + a)+(a + c)
2
b ≤ 6

3
• Trường hợp 2: b ≤ c + a. Khi đó ta có:
f(a, b, c) − f (c, b, a)=b(a −c)(b − c −a)
22
Do đó có thể giả sử c ≥ a. Mặt khác
f(a, b, c) − f (0,b,a+ c)=b(ab −ca − c
2
) ≤ 0 ⇒ f(a, b, c) ≤ b
2
(c + a)+2(a + c)
2
b ≤ 6

3
Vậy ta có điều phải chứng minh Đẳng thức xảy ra khi và chỉ khi c = d =0,b=

3,a=3−


3

 Bài toán : [Đỗ Hoàng Giang]
Cho a,b,c là các số không âm thỏa mãn abc=1. Chứng minh rằng:
a
2
(2a + b)

(1 + a
2
)(1 + b
2
)
+
b
2
(2b + c)

(1 + b
2
)(1 + c
2
)
+
c
2
(2c + a)

(1 + c

2
)(1 + a
2
)

1
2
Lời giải: [Phạm Kim Hùng] Ta đặt
a =
y
x
,b=
z
y
,c=
z
x
Bất đẳng thức trở thành

b
4
(2b
2
+ ac)

(b
2
+ a
2
)(b

2
+ c
2
)

1
2
Sử dụng bất đẳng thức Cauchy-Schawrz ta có
2(a
2
+ b
2
+ c
2
)
2
≥ 2

b
2

(b
2
+ a
2
)(b
2
+ c
2
)+


ac

(b
2
+ a
2
)(b
2
+ c
2
)
Tiếp tục sử dụng bất đẳng thức Cauchy-Schawrz ta có

2b
2

(b
2
+ a
2
)(b
2
+ c
2
) ≤

b
2
(2b

2
+ a
2
+ c
2
)

ac

(b
2
+ a
2
)(b
2
+ c
2
)=


(b
2
c
2
+ a
2
c
2
)(a
2

b
2
+ a
2
c
2
) ≤ 2

a
2
b
2
Công chúng lại ta có điều phải chứng minh.
23
Phần 5 Đại hội Võ lâm - Anh hùng hội ngộ
 Bài toán : [Lê Trung Kiên]
Cho a,b,c là các số không âm. Tìm hằng số k tốt nhất sao cho:
a
2
+ b
2
+ c
2
ab + bc + ca
+ k

a
a + b
+
b

b + c
+
c
c + a

≥ 1+
3k
2
Lời giải [ Bạch Ngọc Thành Công] Khi cho
a
b
=
1
2

1
2

9+6

3+
1
2

3+
1
6

9


3+18
Và với c=0 chúng ta dễ dàng có:k ≤ k
0
=2

9+6

3. Chúng ta sẽ chứng minh với đúng
giá trị này. Đặt k
0
là hằng số thỏa mãn đẳng thức xảy ra. Không mất tính tổng quát giả sử
c = min(a, b, c) Ta có :
LHS − RHS =

a
2
+ b
2
+ c
2
ab + bc + ca
− 1

+ k
0

a
a + b
+
b

b + c
+
c
c + a

3
2

=

1
ab + bc + ca

(a −b)
2
+

1
ab + bc + ca
+
k
0
(a −b)
2(a + b)(b + c)(c + a)

(a − c)(b −c)
Chú ý rằng với mọi số không âm a,b,c chúng ta có đẳng thức sau:
P (a −c)(b −c)(a −b)
2
− P (a −b)

2
(a − c)(b −c)=0
Với P =
−1
a
2
b
2
ta có:

(a − c)(b −c)
a
2
b
2
(a − b)
2
+
(a − b)
2
a
2
b
2
(a − c)(b −c)=0
Đưa bất đẳng thức cần chứng minh về dạng:
f(a, b, c)=M(a −b)
2
+ N (a − c)(b − c)
Trong đó:

M =
1
ab + bc + ca

(a −c)(b −c)
a
2
b
2
=
c
2
(a + b)(a + b −c) −abc
2
a
2
b
2
(ab + bc + ca)
≥ 0
N =
1
ab + bc + ca
+
k
0
(a −b)
2(a + b)(b + c)(c + a)
+
(a − b)

2
ab
Lại có:
N
0
= N(ab + bc + ca)( a + b)(b + c)(c + a)
24
= ab(a + b)+bc(b + c)+ca(c + a)+2abc +
k
0
2
(a
2
b + a
2
c − b
2
c − ab
2
)
+(a + b)(a −b)
2

ab + bc + ca
ab

ab + bc + ca + c
2
ab




k
0
2
+1

+

1 −
k
0
2

+

k
0
2
+1

a
2
c +

1 −
k
0
2


+2abc +(a + b)(a −b)
2

c
a
+
c
b
+1

2
Ta có:
(a + b)

c
a
+
c
b
+1

2
≥ (a + b)

2c
a
+
2c
b
+1


≥ a + b +8c
⇒ N
0
≥ a
3
+ b
3
+
k
0
2
a
2
b −
k
0
2
ab
2
+2abc + a
2
c

k
0
2
+9

+ b

2
c

9 −
k
0
2

− 16abc
= a
3
+ b
3
=
k
0
2
a
2
b −
k
0
2
ab
2
+ a
2
c

9+

k
0
2

+ b
2
c

9 −
k
0
2

−14abc
≥ a
3
+ b
3
+
k
0
2
a
2
b −
k
0
2
ab
2

+

2


9+
k
0
2

9 −
k
0
2

− 14

abc ≥ a
3
+ b
3
+
k
0
2
a
2
b −
k
0

2
ab
2
Ta đặt
S = a
3
+ b
3
+
k
0
2
a
2
b −
k
0
2
ab
2
Dễ dàng kiểm tra rằng
f(t)=t
3
+1+
k
0
2
t
2


k
0
2
t ≥ 0
Trong đó t =
a
b
≥ 0 Từ đó ta có S ≥ 0, bất đẳng thức được chứng minh. Vậy k
max
= k
0

hằng số tốt nhất có thể tìm được

 Bài toán : [Bạch Ngọc Thành Công]
Cho a,b,c là các số không âm. Tìm hằng số k tốt nhất sao cho bất đẳng thức sau là đúng:
a
3
+ b
3
+ c
3
a
2
b + b
2
c + c
2
a
− 1 ≥ k(

a
2
+ b
2
+ c
2
ab + bc + ca
− 1)
Lời giải [Lê Trung Kiên] Ta chỉ cần xét trường hợp a ≥ b ≥ c, k ≥ 0 Cho c =0và a → +∞
suy ra 1 ≥ k Bất đẳng thức cần chứng minh tương đương với:
a
3
+ b
3
+ c
3
− a
2
b − b
2
c − c
2
a
a
2
b + b
2
c + c
2
a

≥ k
a
2
+ b
2
+ c
2
− ab − bc − ca
ab + bc + ca

(a + b)(a −b)
2
+(a + c)(a − c)
2
+(b + c)( b −c)
2
+(a − b)(b − c)(c −a)
a
2
b + b
2
c + c
2
a
25
≥ k
(a −b)
2
+(a −c)
2

+(b − c)
2
ab + bc + ca

(a + b)(a −b)
2
+(a + c)(a − c)
2
+(b + c)( b −c)
2
+(a − b)(b − c)(c −a)
(a −b)
2
+(a −c)
2
+(b − c)
2
≥ k
a
2
b + b
2
c + c
2
a
ab + bc + ca
Ta đặt f(a, b, c)=VT − VP. Ta sẽ chứng minh f(a, b, c) ≥ f(a −c.b − c, 0). Thật vậy
f(a, b, c) ≥ f (a−c.b−c, 0) ⇔
2c(a − b)
2

+2c(a − c)
2
+2c(b − c)
2
(a − b)
2
+(a −c)
2
+(b −c)
2
≥ k
a
2
b + b
2
c + c
2
a
ab + bc + ca
−k(a−c)
⇔ ka +(2−k)c ≥ k
a
2
b + b
2
c + c
2
a
ab + bc + ca
Cái này hiển nhiên đúng khi đó ta chỉ cần chứng minh trong trường hợp c =0,b =1. Khi đó:

a
3
+1
a
2
− 1 ≥ k(
a
2
+1
a
− 1) ⇔ k ≤
a
3
− a
2
+1
a
3
− a
2
+ a
Khi đó ta tìm ra hằng số k tốt nhất là:
k
0
=
2861
909

27
101


x
9
+
58
9x

20
9

2

76x
909

4408
909x
Trong đó:

x =
3

6877 + 909

57

(k
0
≈ 0.808397414 )


 Bài toán : [Đặng Bảo Đức]
Cho a,b,c,d là các số thực. Tìm hằng số k tốt nhất sao cho:
(a
2
+ 3)( b
2
+ 3)( c
2
+ 3)( d
2
+3)≥ k(abc + bcd + cda + dab)
2
Lời giải: [Đặng Bảo Đức) Thử với:
a = b = c = d = t ⇒ (t
2
+4)
4
≥ 16kt
6
⇒ k ≤
(t
2
+3)
4
16t
6
⇒ k ≤
16
9
Ta sẽ chứng minh bất đẳng thức với k =

16
9
Ta đặt
a =
3
x
,b=
3
y
,c =
3
z
,d =
3
t

Tài liệu bạn tìm kiếm đã sẵn sàng tải về

Tải bản đầy đủ ngay
×